Questions tagged [analytic-number-theory]

A beautiful blending of real/complex analysis with number theory. The study involves distribution of prime numbers and other problems and helps giving asymptotic estimates to these.

Filter by
Sorted by
Tagged with
30 votes
3 answers
3k views

Heuristic argument for the prime number theorem?

Here is a bad heuristic argument for the prime number theorem. Let $n$ be a positive integer and assume that PNT holds up to $n$. Then $n$ itself is prime if and only if for each prime $p<n$ the ...
1 vote
1 answer
259 views

GRH and the Euler product

Let $L(\chi, s)$ be the Dirichlet L-Function of a primitive character $\chi$. I believe, if I’m not mistaken, the convergence of the Euler product of $L(\chi, s)$ in the critical strip is known to be ...
68 votes
3 answers
37k views

Yitang Zhang's 2007 preprint on Landau–Siegel zeros

The recent sensational news on bounded gaps between primes made me wonder: what is the status of Yitang Zhang's earlier arXiv preprint On the Landau-Siegel zeros conjecture? If this result is correct, ...
4 votes
2 answers
183 views

Counting integers with k large prime divisors

If $x \ge y \ge 1$ are real numbers and if $k$ is a positive integer, take $\Phi_k(x, y)$ to be the number of integers $\le x$ with exactly $k$ prime factors and no prime factor $\le y$. If $y$ is ...
1 vote
0 answers
70 views

On number of shifted integer solutions to a linear system

Let $M$ be a random diagonally dominant non-singular $3\times 3$ integer matrix $$\begin{bmatrix} m_{11}&m_{12}&m_{13}\\ m_{21}&m_{22}&m_{23}\\ m_{31}&m_{32}&m_{33} \end{...
2 votes
1 answer
554 views

The nontrivial zeros of the zeta function and the prime counting function

The truncated explicit formula has the shape \begin{equation}\label{1} \psi(x) =x-\frac{\zeta^{\prime}(0)}{\zeta(0)}-\sum_{|\rho|\leq T}\frac{x^{\rho}}{\rho}+\sum_{n=1}^{\infty}\frac{x^{-2n}}{2n}+...
8 votes
1 answer
626 views

Absolute convergence of Rankin–Selberg series

Let $\pi$ and $\pi'$ be two general automorphic representations on $\operatorname{GL}(n)$ and $\operatorname{GL}(n')$ over $\mathbb{Q}$. I heard that the Rankin-Selberg $L$-function $L(s,\pi\times\pi')...
9 votes
2 answers
452 views

Distribution $f$ such that (a) $\widehat{f}$ has compact support, (b) $\mathbb{E}(|X|)$ is minimal?

(What follows is motivated by an answer to Fourier optimization problem related to the Prime Number Theorem) Let $f:\mathbb{R}\to [0,\infty)$ be such that (a) $\int_{\mathbb{R}} f(x) dx = 1$, (b) $\...
2 votes
1 answer
210 views

Voronoï summation for cusp forms with characters

In an attempt to solve an unrelated problem, I was led to the task of estimating/bounding from above sums of the form $$\sum_{m=1}^\infty\lambda(m)e\left(-\frac{am}{q}\right)h(m)$$ where $\sum_{m=1}^\...
24 votes
1 answer
2k views

Parity of the multiplicative order of 2 modulo p

Let $\operatorname{ord}_p(2)$ be the order of 2 in the multiplicative group modulo $p$. Let $A$ be the subset of primes $p$ where $\operatorname{ord}_p(2)$ is odd, and let $B$ be the subset of primes $...
2 votes
0 answers
141 views

Expected error term in the distribution of Friedlander-Iwaniec primes

In 1998 John Friedlander and Henryk Iwaniec famously proved the asymptotic formula $$\displaystyle \mathop{\sum \sum}_{a^2 + b^4 \leq x}\Lambda(a^2 + b^4) = \frac{4x^{\frac{3}{4}}}{\pi} \int_0^1 (1 - ...
2 votes
1 answer
271 views

Explicit bounds on number of primes of given size

How many prime numbers of $b$ bits are there? Beyond the prime number theorem, one can give explicit bounds on the number of primes below some integer $n$, or in a given interval. For instance, Rosser ...
3 votes
0 answers
112 views

Bounding number of solutions of a congruence

Let $d$ be a positive integer. Let $f(d,a)$ be the number of values of $x$ in $[1,d]$ such that $$x^{a}\equiv 1\pmod{d}.$$ I wanted to know if for some $0<\epsilon<1$, we can prove the following ...
2 votes
0 answers
193 views

Degree four polynomials with no real roots

Consider a degree four polynomial $$ f = a_4x^4 + a_3x^3 + a_2x^2 + a_1x+ a_0 \in \mathbb{R}[x] $$ with real coefficients. The discriminant $\Delta_f$ of $f$ is a homogeneous polynomials of degree six ...
0 votes
1 answer
124 views

Asymptotic for a sum involving GCD and Euler totient function

Let $\varphi$ denote the Euler's totient function. Is there any reference in literature for the value of sum $$\sum_{\substack{r\le x\\ d\mid r}}\gcd(\phi(d),r)$$ where $d$ is some fixed positive ...
5 votes
0 answers
114 views

Asymptotics for a sum involving GCD and multiplicative order

Let $n$ be a positive integer and $\mathrm{ord}_{n}(a)$ be the least positive integer $d$ such that $n\mid a^{d}-1$. I wanted to know if for some choice of $y=y(x)$, one can obtain asymptotics for ...
2 votes
0 answers
321 views

An approximation for the prime counting function

NOTE: I've edited the question one last time, to be much simpler, in the hopes of getting more responses. SETUP: Let $p_n$ denote the $n$th prime, let $p_x = p_{\lceil x \rceil}$ for all $x > 0$, ...
1 vote
2 answers
267 views

Ask for a reference or a proof of an identity involving a finite sum and the Bernoulli numbers

Let $B_{n}$ for $n\ge0$ denote the Bernoulli number generated by \begin{equation*} \frac{z}{\textrm{e}^z-1}=\sum_{n=0}^\infty B_n\frac{z^n}{n!}=1-\frac{z}2+\sum_{n=1}^\infty B_{2n}\frac{z^{2n}}{(2n)!},...
2 votes
2 answers
271 views

Ask for a proof of an identity involving the product of two Bernoulli numbers

It is well known that the Bernoulli numbers $B_{k}$ for $k\in\{0,1,2,\dotsc\}$ can be generated by \begin{equation*} \frac{z}{\textrm{e}^z-1}=\sum_{k=0}^\infty B_k\frac{z^k}{k!}=1-\frac{z}2+\sum_{k=1}^...
4 votes
0 answers
462 views

Ramanujan's conjecture on modular forms and Riemann hypothesis

I just watched Kannan Soundararajan's talk on the distributions of valus of zeta and $L$-functions at virtual ICM 2022. In his talk, he introduced a theorem on Ramanujan's ternary form $\phi_{1}: x^{2}...
10 votes
0 answers
245 views

A weighted count of Egyptian fraction representations

Previously asked and bountied at MSE: Given a positive rational $q$, let $$\mathsf{E}(q)=\left\{X\in[\mathbb{N}]^{\mathit{fin}}: q=\sum_{x\in X}{1\over x}\right\}$$ be the set of Egyptian fraction ...
10 votes
0 answers
411 views

Are prime numbers among sums of prime numbers distributed as $\frac n{2\ln(n)}$?

Let $(s_n)_{n\in\mathbb N}$ be defined as follows: For $n\in\mathbb N$, $s_n:=2+3+5+\cdots+p_n$ is the sum of the first $n$ prime numbers (e.g.: $s_1=2$, $s_2=5$, $s_3=10$, $s_4=17$, $\ldots$). Let $\...
7 votes
1 answer
1k views

Is it possible to improve on Siegel's theorem for exceptional zeroes?

Let $\chi$ be a real nonprincipal character modulo $q$. Siegel's theorem on exceptional zeroes states that for any $\epsilon >0$ there exists a positive number $C(\epsilon)$ such that, for any real ...
1 vote
1 answer
128 views

Consecutive non-powerful integers

Pair of sequences $\ v_n\ $ and $\ U_n\ $ of integers start as in the following table: [\begin{array}{rrrrrrrrrr} n= & 0 & 1 & 2 & 3 & 4 & 5 & 6 & 7 & \ldots \\ ...
7 votes
0 answers
188 views

The spectrum of the Banach algebra of certain arithmetic functions under Dirichlet convolution

I was thinking about using the tools of functional analysis to study some subring of arithmetic functions under Dirichlet convolution. If I let $D_s$ be the ring of arithmetic functions with finite ...
11 votes
0 answers
236 views

Sums of $\Lambda(n)$ or $\mu(n)$ with hyperbolic-function weights: surprise!

I was leafing through Gradshteyn–Ryzhik in bed yesterday, as one does, and noticed on the last page that the Mellin transforms of several hyperbolic functions have a factor of $\zeta(s-1)$ or $\zeta(s)...
2 votes
1 answer
170 views

Approximating a function by a convolution of given function?

Let $g:\mathbb{R}\to \mathbb{R}$ be a given differentiable function of exponential decay on both sides. Now let us be given a function $f:\mathbb{R}\to \mathbb{R}$, also of exponential decay, if you ...
5 votes
1 answer
825 views

Fastest decay of Fourier transform of function of (one-sided or two-sided) exponential decay

Let $f:\mathbb{R}\to \mathbb{R}$ be a function in $L^2$ satisfying $|f(x)|\ll e^{-a_1 x}$, $a_1>0$, for $x\to \infty$. (Variant: assume as well that $|f(x)|\ll e^{a_2 x}$, $a_2>0$, for $x\to -\...
3 votes
1 answer
902 views

A question on the use of fractional derivatives in Riemann Hypothesis

We already know that Riemann-zeta function on the critical band is defined as follows: $$(1-2^{1-\alpha})\zeta(\alpha) = \sum_{k=1}^{\infty} (-1)^{k+1}k^{-\alpha},\quad \Re(\alpha) \in ]0, 1[ $$ Is ...
4 votes
1 answer
766 views

On Buchstab et al's "forgotten" sieve and the Goldbach conjecture for certain integers

There is a somewhat forgotten sieve-theoretic approach to the Goldbach conjecture, due to Buchstab et al, see e.g. pp.247-248 of R.D. James. On p.247, James defines some function $F$ such that for any ...
15 votes
0 answers
337 views

Do primes of the form $4k+1$ ever lead the greatest prime factor race?

Analogous to Chebyshev's race between primes, I examined the race between primes in the greatest prime factors, GPF, of natural numbers. Similar to the regular prime race, in the GPF race, the ...
2 votes
0 answers
356 views

Classifying solutions of a certain Diophantine Equation

The following question arose from a problem I am working on. Let $N, k$ be positive integers. Consider the Diophantine equation in $a, b, c$: $$ \frac{1}{a} + \frac{N - 1}{b} = \frac{N^k}{c} $$ with ...
2 votes
0 answers
128 views

Eigenfunction of $h\mapsto H(h')|_{[-1,1]}$?

Let $H$ be the Hilbert transform. Is there a continuous, even function $h:\mathbb{R}\to \mathbb{R}$ with support on $[-1,1]$ such that, for some $\lambda\in \mathbb{R}$, $$H(h')(t) = \lambda h(t)$$ ...
6 votes
1 answer
640 views

Fourier optimization problem related to the Prime Number Theorem

Let $\kappa>0$ be given. What is the function $f:\mathbb{R}\to [0,\infty)$ with $\int_\mathbb{R} f(x) dx = 1$ such that $$\int_\mathbb{R} |x| f(x) dx + \kappa \int_{|t|\geq T}\left| \frac{\widehat{...
1 vote
0 answers
139 views

Function involving argument of the Riemann zeta function

When $t$ is an ordinate of a zero of Riemann zeta function, we define \begin{equation} f(t):=\frac{t}{2\pi}\log\left(\frac{t}{2\pi e}\right)+S(t)-\frac{1}{8}+\frac{1}{48 \pi t}+\frac{7}{5760 t^3}+...
0 votes
2 answers
275 views

Counting powerful integers. Lower bounds

Remark:   The upper bounds are perhaps still more interesting; I may address them in another post. PROBLEM:   Find simple (numerically efficient) lower bounds for the number of powerful integers (...
3 votes
1 answer
278 views

Best available bounds for $\pi(Y)-\pi(Y-X)$?

I don't know much (anything) about sieves, but as I read the section on the Selberg upper bound sieve from Greaves's Sieves in Number Theory, there is a theorem 4 which says that If $Y\ge X \ge 2$, ...
5 votes
2 answers
475 views

Optimizing a smoothing function with the Prime Number Theorem in mind

Let $f:[0,\infty)\to \mathbb{R}$ be a function with $f(x)=1$ for $0\leq x\leq 1$. Write $Mf$ for the Mellin transform of $f$. Let $c>0$, $T>10^6$ be constants. We are interested in minimizing ...
4 votes
0 answers
444 views

Question about a paper by Franca and LeClair in analytic number theory

I am reading an article "Transcendental equations satisfied by the individual zeros of Riemann $\zeta$, Dirichlet and modular L-functions" by G. Franca and A. LeClair (2015) see here. The ...
51 votes
6 answers
12k views

What does Mellin inversion "really mean"?

Given a function $f: \mathbb{R}^+ \rightarrow \mathbb{C}$ satisfying suitable conditions (exponential decay at infinity, continuous, and bounded variation) is good enough, its Mellin transform is ...
10 votes
1 answer
458 views

A basic estimate of exponential sums

Demeter in his book "Fourier Restriction, Decoupling, and Applications" (P287) used the following estimate: \begin{equation} \sup_{0\leq n\leq q}\bigg|\sum_{m=0}^n e^{2\pi i\frac{a}{q}m^2}\...
3 votes
0 answers
162 views

Green-Tao's "Polylogarithmic bound for $r_4(N)$"

On P.23 of https://arxiv.org/pdf/1705.01703.pdf, they seemed to suggest that by the non-negativity of $\psi\big(\frac{k}{N}\big)$ for all $k$, $$ K_N(\xi_0 n)\left[1-\cos\bigg(\frac{2\pi\xi_0 n}{p}\...
2 votes
0 answers
171 views

How to best approximate $1/\zeta(s)$ by a finite sum

I would like to approximate $1/\zeta(s)$ for $s=1+it$ by a finite sum: $$\frac{1}{\zeta(1+it)} = \sum_n \frac{\mu(n)}{n} \eta\left(\frac{n}{x}\right) + \epsilon(t)$$ with $\eta$ a function of compact ...
5 votes
0 answers
173 views

On the elementary proof of Dirichlet theorem on arithmetic progressions

In [Cassels, JWS, Rational quadratic forms, p. 333], the autor says: "In fact the elementary proof of Dirichlet's theorem [Selberg (1949)] makes essential use of the existence of genera". In ...
9 votes
0 answers
304 views

Best smoothing for the Prime Number Theorem?

There are plenty of proofs of the Prime Number Theorem with explicit error terms - it actually looks like a rather competitive field (see Remark 1.4 in https://arxiv.org/pdf/2204.02588.pdf). Several ...
3 votes
1 answer
245 views

Successive minima and the basis of lattice

I am able to prove the following two propositions: Recall that the $i$-th successive minimum of $L\in \mathcal L$, denoted $\lambda_i(L)$ is the infimum of the radii of the balls containing $i$-...
2 votes
1 answer
284 views

Overall idea of estimating major arcs in Waring's problem

This is copied from math.SE after a kind comment's suggestion as I am sure people here are very well knowledged in this method :) I am currently reading Vaughan's "The Hardy-Littlewood Method&...
0 votes
0 answers
156 views

Similar problems for the Dedekind psi function than those that are in the literature for the Euler's totient function: reference request or proposals

I know the linked articles from Wikipedia for the Euler's totient function and the Dedekind psi function. I know that are in the literature famous, interesting and difficult problems involving the ...
1 vote
0 answers
114 views

Estimating a sum with a fractional part

Is it possible to pick up the value $\log^2 x<y<x$ to get this expressions simultaneously $$\sum_{d\leq y}\left \{ \frac{x}{d} \right \}\leqslant \frac{y}{\log^{3} y}?$$
1 vote
1 answer
152 views

Existence of a smooth function that approximates a characteristic function of an interval with certain property

Let $N$ be a large integer and $I = [aN, bN]$ for some $0 < a < b < 1$. Denote by $\chi_I(x) = 1$ if $x \in I$, $0$ otherwise. I was wondering if there exists a smooth function $w$ with the ...

1
7 8
9
10 11
59